Difference between revisions of "2024 AMC 10B Problems/Problem 19"
Line 1: | Line 1: | ||
+ | ==Problem== | ||
+ | |||
==Solution 1== | ==Solution 1== | ||
Let's look at zero slope first. All lines of such form will be expressed in the form <math>y=k</math>, where <math>k</math> is some real number. If <math>k</math> is an integer, the line passes through infinitely many lattice points. One such example is <math>y=1</math>. If <math>k</math> is not an integer, the line passes through <math>0</math> lattice points. One such example is <math>y=1.1</math>. So we have <math>2</math> cases. | Let's look at zero slope first. All lines of such form will be expressed in the form <math>y=k</math>, where <math>k</math> is some real number. If <math>k</math> is an integer, the line passes through infinitely many lattice points. One such example is <math>y=1</math>. If <math>k</math> is not an integer, the line passes through <math>0</math> lattice points. One such example is <math>y=1.1</math>. So we have <math>2</math> cases. |
Latest revision as of 01:31, 14 November 2024
Problem
Solution 1
Let's look at zero slope first. All lines of such form will be expressed in the form , where is some real number. If is an integer, the line passes through infinitely many lattice points. One such example is . If is not an integer, the line passes through lattice points. One such example is . So we have cases.
Let's now look at the second case. The line has slope , where and are relatively prime integers. The line has the form . If the line passes through lattice point , then the line must also pass through the lattice point , where are all integers. Therefore, the line can pass through infinitely many lattice points but it cannot pass through exactly or . The line can pass through lattice points, such as . This contributes more cases.
If the line has an irrational slope, it can never pass through more than lattice points. We prove this using contradiction. Let's say the line passes through lattice points and . Then the line has slope , which is rational. However, the slope of the line is irrational. Therefore, the line can pass through at most lattice point. One example of this is . This line contributes final cases.
Our answer is therefore .
~lprado
See also
2024 AMC 10B (Problems • Answer Key • Resources) | ||
Preceded by Problem 18 |
Followed by Problem 20 | |
1 • 2 • 3 • 4 • 5 • 6 • 7 • 8 • 9 • 10 • 11 • 12 • 13 • 14 • 15 • 16 • 17 • 18 • 19 • 20 • 21 • 22 • 23 • 24 • 25 | ||
All AMC 10 Problems and Solutions |
The problems on this page are copyrighted by the Mathematical Association of America's American Mathematics Competitions.